Find x

x³ + 3x - 14 = 0

x³ + x² - x² - x + 4x + 4 = 18
x²(x + 1) - x(x + 1) + 4(x + 1) = 18
(x + 1)(x² - x + 4) = 18
x² - x + 4 = 18/(x + 1)
x² - x + 4 - 6 = 18/(x + 1) - 6
x² - x - 2 = 18/(x + 1) - 6
(x - 2)(x + 1) = (18 - 6(x + 1))/(x + 1)
(x - 2)(x + 1) = (18 - 6x - 6)/(x + 1)
(x - 2)(x + 1) = (12 - 6x)/(x + 1)
(x - 2)(x + 1) = (-6(x - 2))/(x + 1)
x + 1 = (-6(x - 2))/(x + 1)(x - 2)
x + 1 = -6/(x + 1)
(x + 1)² = -6
x² + 2x + 8 = 0
x = (-b +- √(b² - 4ac))/2a
x = (-2 +- √(4 - 32))/2
x = (-2 +- √(-28)/2
x = (-2 +- i√28)/2​

Something's wrong.​

Answers

Answer 1

[tex]\implies {\blue {\boxed {\boxed {\purple {\sf { \: x = - 1 \: + \: i \sqrt{6} \:(or) \: \: x = - 1 \: -\: i \sqrt{6} }}}}}}[/tex]

And

[tex]\implies {\blue {\boxed {\boxed {\purple {\sf {x\:=\:2}}}}}}[/tex]

[tex]\large\mathfrak{{\pmb{\underline{\red{Step-by-step\:explanation}}{\red{:}}}}}[/tex]

[tex] \: {x}^{3} + 3x - 14 = 0[/tex]

➺[tex] \: {x}^{2} (x + 1) - x(x + 1) + 4(x + 1) = 18[/tex]

➺[tex] \: (x + 1)( {x}^{2} - x + 4) = 18[/tex]

➺[tex] \: {x}^{2} - x + 4 = \frac{18}{(x + 1)} [/tex]

➺[tex] \: {x}^{2} - x + 4 - 6 = \frac{18}{(x + 1)} - 6[/tex]

➺[tex] \: (x - 2)(x + 1) = \frac{18 - 6(x + 1)}{(x + 1)} [/tex]

➺[tex] \: (x - 2)(x + 1) = \frac{18 - 6x - 6}{(x + 1)} [/tex]

➺[tex] \: (x - 2)(x + 1) = \frac{12 - 6x}{(x + 1)} [/tex]

➺[tex] \: (x - 2)(x + 1) = \frac{ - 6(x - 2)}{(x + 1)} [/tex]

➺[tex] \: (x + 1 )² = \frac{ - 6(x - 2)}{(x + 1)(x - 2)} [/tex]

➺[tex] \: (x + 1)² = \frac{ - 6}{(x + 1)} [/tex]

[tex]\sf\pink{Error\:corrected\:here. }[/tex]

➺[tex] \: {x}^{2} + 2x + 1 = - 6[/tex]

➺[tex] \: {x}^{2} + 2x + 7 = 0[/tex]

By quadratic formula, we have

➺[tex] \: x = \frac{ - b± \sqrt{ {b}^{2} - 4ac} }{2a} [/tex]

➺[tex] \: x = \frac{ - 2± \sqrt{ {2}^{2} - 4.1.7} }{2 \times 1} [/tex]

➺[tex]x = \frac{ - 2± \sqrt{ - 24} }{2 } [/tex]

➺[tex] \: x = \frac{ - 2± \sqrt{ - 1 \times 4 \times 6} }{2} [/tex]

➺[tex] \: x = \frac{ - 2± \sqrt{ - 1} \times \sqrt{4} \times \sqrt{6} }{2} [/tex]

➺[tex] \: x = \frac{ - 2 \: ± \: i \times 2 \times \sqrt{6} }{2} [/tex]

➺[tex] \: x = \frac{ - 2 \: ± \:i \: 2 \sqrt{6} }{2} [/tex]

➺[tex] \: x = \frac{ 2 \: ( - 1 \: ± \: i \: \sqrt{6}) }{2} [/tex]

➺[tex] \: x = - 1 \: ± \: i \sqrt{6} [/tex]

Therefore, the two values of [tex]x[/tex] are ([tex] \: - 1 \: + \: i \sqrt{6}[/tex]) and ([tex] \: - 1 \: -\: i \sqrt{6}[/tex]).

Let us look at another method.

[tex]x[/tex]³ + 3 [tex]x[/tex] - 14 = 0

➼ [tex]x[/tex]³ + 3 [tex]x[/tex] = 14

➼ [tex]x[/tex] ( [tex]x[/tex]² + 3 ) = 14

Factors of 14 = 1, 2, 7 and 14.

a) Substituting [tex]x\:=\:1[/tex], we have

➼ 1 ( 1 + 3 ) ≠ 14

➼ 1 x 4 ≠ 14

➼ [tex]\boxed{ 4\: ≠ \:14 }[/tex]

b) Substituting [tex]x\:=\:2[/tex], we have

➼ 2 ( 2² + 3 ) = 14

➼ 2 ( 4 + 3 ) = 14

➼ 2 x 7 = 14

➼ [tex]\boxed{ 14 \:= \:14 }[/tex]

c) Substituting [tex]x\:=\:7[/tex], we have

➼ 7 ( 7² + 3 ) ≠ 14

➼ 7 ( 49 + 3 ) ≠ 14

➼ 7 x 52 ≠ 14

➼ [tex]\boxed{ 364\: ≠ \:14 }[/tex]

d) Substituting [tex]x\:=\:14[/tex], we have

➼ 14 ( 14² + 3 ) ≠ 14

➼ 14 x 199 ≠ 14

➼ [tex]\boxed{ 2786\: ≠ \:14 }[/tex]

Hence, our only real solution is 2.

[tex]\large\mathfrak{{\pmb{\underline{\orange{Mystique35 }}{\orange{❦}}}}}[/tex]


Related Questions

it cost $246 to turf a lawn of 43m ² . how much will it cost to turf a lawn of 61m square 2 .​

Answers

Answer:

$348.96

Step-by-step explanation:

246÷43=5.72

61-43=18

18×5.72=102.96

102.96+246=348.96

Four students want to have their picture taken together. They will stand side-by-side for the picture. In how many different ways can the four students be arranged to take a picture?

Show your work, please :')

Answers

Answer:

24 ways

Step-by-step explanation:

This is the permutation of 4:

4P4 = 4! = 1*2*3*4 = 24 ways

[tex]\huge\qquad \mathbb{\fcolorbox{red}{lavenderblush}{✰Answer}}[/tex]

✶⊶⊷⊶⊷❍❁❥❀❥❁❍⊶⊷⊶⊷✶

Four students want to have their picture taken together. They will stand side-by-side for the picture. In how many different ways can the four students be arranged to take a picture

so we have to find the permutation of 4

4×3×2×124

.°. In 24 different ways can the four students be arranged to take a picture

A medical device company knows that the percentage of patients experiencing injection-site reactions with the current needle is 11%.

What is the probability that an injection-site reaction occurs for the first time on the 6th patient of the day?

0.0001
0.0614
0.3685
0.4970

Answers

Answer:

0.4970

Step-by-step explanation:

I might be wrong

The probability that an injection-site reaction occurs for the first time on the 6th patient of the day will be 0.4970. Then the correct option is D.

What is probability?

Its fundamental concept is that someone will nearly surely occur. The proportion of positive events in comparison to the total of occurrences.

Then the probability is given as,

P = (Favorable event) / (Total event)

A medical device company knows that the percentage of patients experiencing injection-site reactions with the current needle is 11%.

The probability that an injection-site reaction occurs for the first time on the 6th patient of the day is given as,

P = (1 - 0.11)⁶

P = (0.89)⁶

P = 0.4970

The probability that an injection-site reaction occurs for the first time on the 6th patient of the day will be 0.4970. Then the correct option is D.

More about the probability link is given below.

https://brainly.com/question/795909

#SPJ2

Can anyone help me with this question

Answers

Answer:

50%

Step-by-step explanation:

If angle f is 50 degrees and cuts it to 75 degrees

Find the difference between the two numbers

75-50 = 25

Divide by the correct value

25/50 = 1/2

Change to percent form

50%

Suppose that g(x)= f(x)+ 6. Which statement best compares the graph of g(x) with the graph of f(x)?

A. The graph of g(x) is the graph of f(x) shifted 6 units down.

B. The graph of g(x) is the graph of f(x) shifted to the right.

C. The graph of g(x) is the graph of f(x) shifted 6 units to the left.

D. The graph of g(x) is the graph of f(x) shifted 6 units up.

Answers

Answer:

D

Step-by-step explanation:

The + 6 moves it up 6 units.

The correct answer is (D) "The graph of g(x) is the graph of f(x) shifted 6 units up."

What is the function?

A relationship between a group of inputs and one output is referred to as a function. In plain English, a function is an association between inputs in which each input is connected to precisely one output. A domain, codomain, or range exists for every function. Typically, f(x), where x is the input, is used to represent a function.

When we add a constant to a function, such as in the case of g(x) = f(x) + 6, it will shift the graph of f(x) upward by 6 units.

This is because, for any value of x, the value of f(x) will be added to 6, resulting in a vertical shift of the entire graph.

Option (A) is incorrect because adding 6 to f(x) would shift the graph up, not down.

Option (B) is incorrect because adding a constant to a function does not cause it to shift horizontally.

Option (C) is incorrect because adding 6 to f(x) would shift the graph right, not left.

D. The graph of g(x) is the graph of f(x) shifted 6 units up. Adding a constant term to a function will shift the graph of the function vertically. In this case, adding 6 to f(x) will shift the graph of f(x) upward by 6 units, resulting in the graph of g(x).

Learn more about function here:

https://brainly.com/question/29633660

#SPJ7

After a new product is launched the cumulative sales S(t) (in $1000) t weeks after launch is given by:

S(t) = 72/1 + 9e^-0.36t

Required:
a. Determine the cumulative amount in sales 3 weeks after launch.
b. Determine the amount of time required for the cumulative sales to reach $70,000.
c. What is the limiting value in sales?

Answers

Answer:

$17.750 ; 15.979 ; 72

Step-by-step explanation:

Given that :

Cummulative sales, S(t) is represented by the equation :

S(t) = 72/(1 + 9e^-0.36t)

Cummulative sales after 3 weeks :

Put t = 3 in the equation, as t = time after launch

S(3) = 72/(1 + 9e^-0.36(3))

S(3) = 72 / (1 + 9e^-1.08)

S(3) = 72 / (1 +3.0563597)

S(3) = 72 / 4.0563597

S(3) = 17.749905 = $17.750 thousands

Amount of time required for sales to reach 70000

S(t) = 72/(1 + 9e^-0.36t)

S(t) = 70

70 = 72/(1 + 9e^-0.36t)

70 * (1 + 9e^-0.36t) = 72

(1 + 9e^-0.36t) = 72 / 70

1 + 9e^-0.36t = 1.0285714

9e^-0.36t = 1.0285714 - 1

9e^-0.36t = 0.0285714

e^-0.36t = 0.0285714 / 9

e^-0.36t = 0.0031746

Take the In of both sides ;

In(e^-0.36t) = In(0.0031746)

-0.36t = - 5.752573

t = - 5.752573 / - 0.36

t = 15.979

About 16 weeks

The limiting value in sales :

Take the limit as t - - > ∞

S(t - - > ∞) = 72/(1 + 9e^-0.36t)

Put t = 0

S(0) - - > 72 / (1 + 0)

72 / 1

= 72

C
What is the equation of the line that is parallel to the line y - 1 = 4(x + 3) and passes through the point (4, 32)?
SER
O y=-*x+33
y=-*x+36
O y =
O y = 4x - 16
O y = 4x + 16
Mark this and retum
Save and Exit
Nex
Submit

Answers

9514 1404 393

Answer:

  y = 4x + 16

Step-by-step explanation:

You can replace the point coordinates in the given point-slope form to get ...

  y - 32 = 4(x -4)

Then rearrange to put this into slope-intercept form.

  y = 4x -16 +32 . . . . eliminate parentheses, add 32

  y = 4x +16 . . . . . . . . simplify

The football and soccer teams at Juan's middle school are selling movie tickets. He wants to determine the average
number of tickets sold at the school. First, he surveyed eight random players on the football team. Then, he surveyed
every third athlete on the football and soccer team. Which sampling technique should produce a more representative
sample?
The first sampling method, surveying eight random football players, is the most representative.
O Neither sample will be representative.
Both samples should be exactly the same.
The second sampling method, survey.ing the both teams, is the most representative.

Answers

Step-by-step explanation:

both samples should be exactly the same

If 21% of kindergarten children are afraid of monsters, how many out of
each 100 are afraid?

Answers

Answer:

The appropriate answer is "21".

Step-by-step explanation:

Given:

Afraid percentage,

p = 21%

or,

  = 0.21

Sample size,

n = 100

As we know,

⇒ [tex]X=np[/tex]

By putting the values, we get

        [tex]=0.21\times 100[/tex]

        [tex]=21[/tex]

To estimate the difference we need four averages for the categorized groups i.e., control group before change, control group after change, treatment group before change and treatment group after change.

a. True
b. False

Answers

Answer:

b. False

Step-by-step explanation:

In a research study, when a researcher wants to find the impact of a new treatment, then the researcher randomly divides the the study participants into two groups. The groups are :

-- control group

-- treatment group

The control group is a group that is used to establish the cause-and-effect relationship by making the effect of an independent variable isolate. It receives no treatment or some standard treatment for the which the effect is already known.

The treatment group receives the treatment for which the effect the researcher is interested in.

Thus the averages of the four categorized groups are not required for estimating the difference.

Therefore, the answer is FALSE.

Một đề thi trắc nghiệm có 10 câu, mỗi câu có 4 phương án trả lời và chỉ có một đáp án đúng. Một sinh viên trả lời một cách ngẫu nhiên, xác suất để sinh viên được 5 điểm là

Answers

Câu trả lời:

0,0584

Giải thích từng bước:

Câu hỏi đưa ra đáp ứng điều kiện cần thiết cho phân phối xác suất nhị thức:

Số câu hỏi, số lần thử, n = 10

Xác suất, p = 1 / số lựa chọn = 1/4 = 0,25

q = 1 - p = 1 - 0,25 = 0,75

Tính xác suất để sinh viên đó được 5 điểm, x = 5;

Gợi lại:

P (x = x) = nCx * p ^ x * q ^ (n-x)

P (x = 5) = 10C5 * 0,25 ^ 5 * 0,75 ^ 5

P (x = 5) = 252 * 0,25 ^ 5 * 0,75 ^ 5

P (x = 5) = 0,0584

I'm not good with ordering fractions from smallest to largest. Can help anyone help with this problem?

Answers

5/8,4/6,3/4,1/2 this is the answer without simplifying

The length of a rectangle is four times its width.
If the area of the rectangle is 100 yd”, find its perimeter.

Answers

Answer: 50yd

Step-by-step explanation:

We know that the area of any rectangle is length times width.  The perimeter is the sum of twice the length and twice the width.

Let width = x

Let length = 4x

Area = 100m2

Next, we can write an equation using these variables and formula for area.

4x2 = 100

x2 = 25

x = -5      and        x = 5

Since the dimensions cannot be negative, we accept the positive value:

x = 5

Next, we can substitute this value of x into the variables.

width = 5 m

length = 20 m

Finally, we can find the perimeter by plugging in these dimensions into the perimeter formula,

Perimeter = 2(5 m) + 2(20 m)

              = 10 m + 40 m

              = 50 m  

Look at the illustration.
What is WX?

Answers

Answer:

O  0.5 units

Step-by-step explanation:

so the first thing we have to do is to calculate for the dilation factor. Taking point G as the reference point, we can see that the distance of point G from rectangle W'X'Y'Z is 1.5 while the distance from rectangle WXYZ is (1.5 + 7.5) = 1.5 / 9 = 1/6

Since WX has an initial measure of 3 units, therefore the measure of W'X' is:

W'X' = 3 units *(1/6) = 0.5 units

If f(x) = x -2 and g(x) = 2x – 6, then g(4)/f(3) =​

Answers

Answer:

Step-by-step explanation:

(2×4-6)/(3-2)=2

Answer:

[tex]{ \tt{f(x) = x - 2}} \\ { \bf{f(3) = 3 - 2 = 1}} \\ \\ { \tt{g(x) = 2x - 6}} \\ { \bf{g(4) = 2(4) - 6 = 2}} \\ \\ { \boxed{ \tt{ \frac{g(4)}{f(3)} = \frac{2}{1} = 2}}}[/tex]

What is the value of z2 if 3z1 = 6 + 18i and z1 + z2 = 5 + 9i?

Answers

Answer:

z2=-3-3i

Step-by-step explanation:

3z1+0z2=6+18i

z1+z2=5+9i (×3)

3z1+3z2=15+27i

3z1+0z2=6+18i (-) =

-3z2=9+9i

z2=-3÷(9+9i)

z2=-3-3i

Please help me with this question

Answers

C. None from the above

20 POINTS MATH PROBLEM

Answers

Answer:

D. x=36

Step-by-step explanation:

3x-5=103

3x=103+5

3x=108

x=36

Find the value of PR if Q is between P and R
when PQ=25, PQ=2x+1, and QR=x.
X
ат
X=8

Answers

Answer:

[tex]PR = 37[/tex]

Step-by-step explanation:

Given

[tex]PQ = 25[/tex]

[tex]PQ = 2x +1[/tex]

[tex]QR = x[/tex]

Required

PQ

Since Q is between the given points, then:

[tex]PR = PQ + QR[/tex]

This gives:

[tex]PR = 2x + 1 + x[/tex]

Collect like terms

[tex]PR = 2x + x+ 1[/tex]

[tex]PR = 3x + 1[/tex]

Next, solve for x

We have:

[tex]PQ = 25[/tex]

[tex]PQ = 2x +1[/tex]

This gives:

[tex]2x + 1 = 25[/tex]

Collect like terms

[tex]2x = 25 -1[/tex]

[tex]2x = 24[/tex]

Divide by 2

[tex]x = 12[/tex]

So:

[tex]PR = 3x + 1[/tex]

[tex]PR = 3 * 12 + 1[/tex]

[tex]PR = 37[/tex]

Simplify log2 20-log2 30.​

Answers

Answer:

[tex] log_{2}(20) - log_{2}(30) \\ log_{2}(20 \div 30) \\ log_{2}(0.667) [/tex]

hope this helps you

Brainliest appreciated

log1(20)-log2(30)

log2 (20÷30)

log2=0.667

Sketch the region enclosed by the given curves and calculate its area.
y=4-x^2 ,y=0

The answer is 32/3. But how do I get to that answer?

Answers

Answer:

Step-by-step explanation:

1.) we need to find the bounds of integration which is just the points of intersection

here is it (-2,0) and (2,0)

which means we will integrate from -2 to 2

next, we take the upper equation and subtract that from the lower one

kind of confusing but it would look like (sketch it out if you're not sure)

(4-x²)-0= 4-x²

then we can integrate

[tex]\int\limits^2_{-2} {4-x^2} \, dx =4x-\frac{x^3}{3}|_{-2}^{2}=(4*(2)-\frac{2^3}{3})-(4(-2)-\frac{-2^3}{3})=5.333333-(-5.3333333)= 10.666666667=\frac{32}{3}[/tex]

Determine which value best approximates the length of the arc represented by the integral ∫_0^1 √1 + [d/dx(4/x+1)]² dx.
(Make your selection on the basis of a sketch of the arc and not
by performing any calculations.)
(a) 10
(b) -5
(c) 2
(d) 4
(e) 1

Answers

Answer:

Option C

Step-by-step explanation:

From the question we are told that:

Length of arc integral

[tex]l=\int_0^1 \sqrt{1 + [\frac{d}{dx}(\frac{4}{x+1})]^2 dx}[/tex]

The Sketch is attached below

From the Graph

Approximation gives length of arc as

[tex]l=\sqrt{5}[/tex]

[tex]l=2[/tex]

Option C

Jack is 2 years older than his sister jill. The sum of their ages is 24 years. If jack is x years, find x

Answers

Answer:

x = 13

Jack is 13 years old and Jill is 11 years old.

====================================================

Explanation:

x = Jack's age

x-2 = Jill's age, since she is 2 years younger than Jack

Add up the ages and set the sum equal to 24 to solve for x

x+(x-2) = 24

2x-2 = 24

2x = 24+2

2x = 26

x = 26/2

x = 13

Jack is 13 years old and his sister is x-2 = 13-2 = 11 years old

Check: 13+11 = 24, so the answer is confirmed.

35 of 39
Complete the item by determining if the equation is true or false,
5x 3 1/2 = 3 x 5 1/2
0 17.5 = 16.5 (false)
O 17.5 = 17.5 (true)
O 18,5 = 18,5 (true)
O 18.5 = 19.5 (false)

Answers

Step-by-step explanation:

5x 3 1/2 = 3 x 5 1/2 is false

The upwards acceleration of a small rocket at time t s is given by a = 16 − 1.5t. The rocket is subject to this
acceleration for 3 seconds. Given that it starts from rest at t = 0, calculate the height reached by the rocket in
this time

Answers

Answer:

11.5

Step-by-step explanation:

plug 3 in for x

then solve

Find the area of the rectangle if the perimeter is 52 cm

Answers

Answer:

No solution is possible from the information provided

Step-by-step explanation:

Add the following fractions. See the image below

Answers

Answer:

[tex]\dfrac{19}{840}[/tex]

Step-by-step explanation:

The given fraction is:

[tex]\dfrac{1}{168}+\dfrac{3}{180}[/tex]

We need to solve it.

The LCM of 168 and 180 is 2520.

So,

[tex]\dfrac{1}{168}+\dfrac{3}{180}=\dfrac{15+3\cdot14}{2520}\\\\=\dfrac{19}{840}[/tex]

So, the required answer is equal to [tex]\dfrac{19}{840}[/tex].

What is the slope intercept form of the equation of the line shown below

Answers

Answer:

[tex]y=\frac{4}{3}x-4[/tex]

Step-by-step explanation:

----------------------------------------

The slope-intercept form formula is: [tex]y=mx+b[/tex]

The [tex]m[/tex] stands for the slope and the [tex]b[/tex] stands for the y-intercept.

By looking at the graph, I can figure out that the y-intercept is -4 because y-intercept is where the lines cross the y-axis and in this graph, the line crosses the y-axis at (0,-4).

The slope is [tex]\frac{4}{3}[/tex] because to get to the ordered pair (3,0), from (0,-4), you would have to go up 4 and over 3 to the right so it's [tex]\frac{4}{3}[/tex]

So now, if we insert the values in the formula, it would be [tex]y=\frac{4}{3}x-4[/tex]

----------------------------------------

Hope this is helpful.

Answer:

y = 4/3x -4

Step-by-step explanation:

First find the slope

m = ( y2-y1)/(x2-x1)

    = ( 0 -  -4)/( 3 - 0)

   = (0+4)/( 3-0)

   = 4/3

The y intercept is -4

The slope intercept form of the equation is

y = mx+b  where m is the slope and b is the y intercept

y = 4/3x -4

help pleaseeee it’s timed!!!

Answers

Answer:

C

Step-by-step explanation:

The solution triangle is attached below :

Tonobtinnthe Angle formed, θ; we apply trigonometry ;

Using ;

Cos θ = Adjacent / hypotenus

Cos θ = 4 / 7

θ = Cos^-1(4/7)

θ = 55.15°

θ = 55°

A cyclist travels at an average speed of 18 mph for 2 hours.
How far did she travel in miles?

Answers

Answer:

36

Step-by-step explanation:

18 miles per hour so for 2 hours which is double, she should have traveled 36 miles.

Good Luck!

Other Questions
15 points please explain it to me add the answer and the work since Im so confused on when to subtract missing lengths where and when A country that has a highly specialized economy is most likely to experiencewhich problem?A. It does not produce all the essential goods its people need.B. It cannot convince other countries to make trade agreements.OC. It produces too many different products at low efficiency.D. It must pay higher tax rates to international trade organizations.SUBMI Please help me kitty Joey has 2/3 of a gallon of ice cream. He divides the ice cream for his friends so that 1/9 of a gallon is in each bowl. How many bowls will he use?A. 6B. 1/6C. 27D. 1/9 which word from abbey's passage best demonstrates the ad hominem fallacy in his argument? what is the suffix fright Difference between radiation conduction and convection detailed with examples. Ill give brainliest. Here is the height, in centimeters, of 20 flowers in the school garden.Based on the data, which of these can be considered a typical height for these flowers? A. 5 centimeters B. 25 centimeters C. 30 centimeters D. 110 centimeters(the numbers in the picture go from 5 to 110) If x=28, find the measures of angles 1, 2, and 3. ?? I got 5 minutes left, please help. Three resistors (16 ohm, 16 ohm and 8 ohm) are connected in parallel. The equivalentresistance (Re) For the function F defined by F(x) = x2 2x + 4, find F(| 4|)A. 28B. 12C. 8D.4 9- Which type of view is not present in MSPowerPoint?(A) Extreme animation(B) Slide show(C) Slide sorter(D) Normal Write an equation and solve for x. Show all your work. realiza una autobiografa aplicando las normas apa mnimo una pgina incluir gustos deportes favoritos y todo lo que tenga que ver contigo Can someone give me the answer ?!? 10.)A child-sized swimming pool can be modeled by a cylinder. The pool has a diameter of 6 1/2 feet and a height of 12 inches. The pool is filled with water to 2/3 of its height. Determine and state the volume of the water in the pool, to the nearest cubic foot. One cubic foot equals 7.48 gallons of water. Determine and state, to the nearest gallon, the number of gallons of water in the pool. [tex]\sqrt{75x^{7} }[/tex] simplified Which is the interquartile range of the data set: 8, 9, 10, 11, 12?A. 4b. 4.5C. 6D. 3 A flexible-budget variance is $800 favorable for unit-related costs. This indicates that costs were: _____________a. $800 more than the master budget b. $800 less than for the planned level of activity c. $800 more than standard for the achieved level of activity d. $800 less than standard for the achieved level of activity